Summer is a great time to explore cool problems to keep your skills sharp!  Schedule a class today!

G
Topic
First Poster
Last Poster
AMC 12 How to Study
krishiam   1
N Yesterday at 2:03 PM by Alex-131
Hi, I am in my junior year and want to study for the upcoming AMC 12, but I'm not sure. I have done AOPS Geometry and Intermediate Algebra, but I forgot everything, and I have just a strong foundation in high school math and a basic understanding of probability, and know just the basic stuff needed. For reference, I scored like somewhere in the 70s AMC 10 last year. How should I prepare? Is grinding practice problems enough to qualify for AIME?
1 reply
krishiam
Yesterday at 12:35 PM
Alex-131
Yesterday at 2:03 PM
funny log fraction
OronSH   13
N Jun 7, 2025 by SomeonecoolLovesMaths
Source: 2024 AMC 12B #8
What value of $x$ satisfies \[\frac{\log_2x\cdot\log_3x}{\log_2x+\log_3x}=2?\]
$
\textbf{(A) }25\qquad
\textbf{(B) }32\qquad
\textbf{(C) }36\qquad
\textbf{(D) }42\qquad
\textbf{(E) }48\qquad
$
13 replies
OronSH
Nov 13, 2024
SomeonecoolLovesMaths
Jun 7, 2025
Complex roots solution clarification
yingkai_0_   2
N May 20, 2025 by yingkai_0_
Source: https://artofproblemsolving.com/wiki/index.php/2020_AMC_12B_Problems/Problem_17
on solution 1, how do we get to the conclusion that if we have another root w, then we must have $we^{\frac{2pi}{3}}$ and $we^{\frac{4pi}{3}}$, respectively? can't u have 2 other roots like 2 and 3, for example where they are not complex or irrational?

please help :(
2 replies
yingkai_0_
May 20, 2025
yingkai_0_
May 20, 2025
Harmonic Mean
Happytycho   4
N May 16, 2025 by elizhang101412
Source: Problem #2 2016 AMC 12B
The harmonic mean of two numbers can be calculated as twice their product divided by their sum. The harmonic mean of $1$ and $2016$ is closest to which integer?

$\textbf{(A)}\ 2 \qquad
\textbf{(B)}\ 45 \qquad
\textbf{(C)}\ 504 \qquad
\textbf{(D)}\ 1008 \qquad
\textbf{(E)}\ 2015 $
4 replies
Happytycho
Feb 21, 2016
elizhang101412
May 16, 2025
Centroids form Equilateral Triangle
Generic_Username   22
N May 5, 2025 by Tetra_scheme
Source: 2019 AMC 12B #25
Let $ABCD$ be a convex quadrilateral with $BC=2$ and $CD=6.$ Suppose that the centroids of $\triangle ABC,\triangle BCD,$ and $\triangle ACD$ form the vertices of an equilateral triangle. What is the maximum possible value of the area of $ABCD$?

$\textbf{(A) } 27 \qquad\textbf{(B) } 16\sqrt3 \qquad\textbf{(C) } 12+10\sqrt3 \qquad\textbf{(D) } 9+12\sqrt3 \qquad\textbf{(E) } 30$
22 replies
Generic_Username
Feb 14, 2019
Tetra_scheme
May 5, 2025
Real Analysis on AMC
Generic_Username   50
N Apr 28, 2025 by ESAOPS
Source: 2019 AMC 12B #22
Define a sequence recursively by $x_0=5$ and
\[x_{n+1}=\frac{x_n^2+5x_n+4}{x_n+6}\]for all nonnegative integers $n.$ Let $m$ be the least positive integer such that
\[x_m\leq 4+\frac{1}{2^{20}}.\]In which of the following intervals does $m$ lie?

$\textbf{(A) } [9,26] \qquad\textbf{(B) } [27,80] \qquad\textbf{(C) } [81,242]\qquad\textbf{(D) } [243,728] \qquad\textbf{(E) } [729,\infty]$
50 replies
Generic_Username
Feb 14, 2019
ESAOPS
Apr 28, 2025
Is my approach right?
cowcheese   1
N Apr 15, 2025 by YauYauFilter
Source: AMC12B 2019 p11
How many unordered pairs of edges of a given cube determine a plane?

a. 12. b. 28 c. 36 d. 42. e. 66

I ended up with the right answer. However, no solutions resembled the way I did it, which is why I'm skeptical. After a few failed attempts with other ways of solving (didn't get an answer in the answer options), I decided to use complementary counting, and counted invalid pairs (pairs that don't satisfy the condition given by the problem). To do that I drew out the cube and found for each side, there are 4 sides that will make an invalid pair with it. Because there are 12 sides to a cube, I did 12*4. But, each distinct pair in that 12*4 is counted twice. So, dividing 12*4 by 2 will give 24 invalid pairs. Now, for total pairs, it is 12 choose 2, which is 66. 66-24 = 42. Hence, the answer is 42.

So, is there anything wrong with the way I did it? Is there a specific reason this wasn't a posted solution?

Thanks!
1 reply
cowcheese
Apr 14, 2025
YauYauFilter
Apr 15, 2025
9 What motivates you
AndrewZhong2012   70
N Mar 20, 2025 by pingpongmerrily
What got you guys into math? I'm asking because I got ~71 on the AMC 12B and 94.5 on 10A last year. This year, my dad expects me to get a 130 on 12B and 10 on AIME, but I have sort of lost motivation, and I know these goals will be impossible to achieve without said motivation.
70 replies
AndrewZhong2012
Feb 22, 2025
pingpongmerrily
Mar 20, 2025
AMC 12 Question
sadas123   12
N Mar 19, 2025 by jb2015007
Hello! I am a 6th grader this year about to become 7th grade next year. I was wondering if I should take the AMC 12 next year because I think I am ready for it, I was thinking to do AMC 10 A and AMC 12 B, do you think it is a good idea? Here are the courses I finished and now I am working on:

Finished:
1. Intro Algebra
2. Intro Number Theory
3. Intro Counting and Probability
4. Volume 1

Working on:
1. Intermdiate Counting and Probability
2. Three Year Mathcounts Marathon

Upcoming:
1. Intro Geomtery (Next Month)
2. Intro to Alg (May)
3. Pre-calc (Summer)
4. Volume 2???

Stats for AMC 12 (Mocked):

1. AMC 12 A 2024: 100.5
2. AMC 12 B 2024: 105
3. AMC 12 A 2023: 96

The reason why I sometimes I get 100+ is because sometimes I know how to do the first step of the problem but the last step I have to kind of infrence but still i know how to do the problem.
12 replies
sadas123
Mar 18, 2025
jb2015007
Mar 19, 2025
Accounting for an overcount
WhaleVomit   61
N Mar 17, 2025 by SomeonecoolLovesMaths
Source: 2017 AMC 10B #17, AMC 12B #11
Call a positive integer monotonous if it is a one-digit number or its digits, when read from left to right, form either a strictly increasing or a strictly decreasing sequence. For example, 3, 23578, and 987620 are monotonous, but 88, 7434, and 23557 are not. How many monotonous positive integers are there?

$\textbf{(A)} \text{ 1024} \qquad \textbf{(B)} \text{ 1524} \qquad \textbf{(C)} \text{ 1533} \qquad \textbf{(D)} \text{ 1536} \qquad \textbf{(E)} \text{ 2048}$
61 replies
WhaleVomit
Feb 16, 2017
SomeonecoolLovesMaths
Mar 17, 2025
a